1
$\begingroup$

Consider parameterized linear programming $V(\theta) = \max_x \langle c(\theta),x\rangle$ s.t. $A(\theta)x\leq b(\theta)$, $x\geq 0$. Let's also assume $c,A,b$ are infinitely differentiable with respect to $\theta$.

It is known (the envelope theorem) that under suitable conditions (one sufficient condition is that the set of binding constraints does not change for all $\theta\in B_\delta(\theta_0)=\{\theta: \|\theta-\theta_0\|\leq\delta\}$ for some $\delta>0$) the objective $V$ is differentiable in $\theta$ for $\theta\in \mathrm{Int}B_\delta(\theta_0)$ and furthermore $$ \nabla_\theta V(\theta) = \langle\nabla c(\theta), x^*(\theta)\rangle-\langle\lambda^* (\theta) ,\nabla b(\theta) -\partial_\theta A(\theta)x^*(\theta)\rangle, $$ where $x^*(\theta)$ is the optimal solution at $\theta$ and $\lambda^*(\theta)$ is the optimal Lagriangian multipler for all constraints $A(\theta)x\leq b(\theta)$.

My question is: under the condition that the set of binding constraints does not change for all $\theta\in B_\delta(\theta_0)=\{\theta: \|\theta-\theta_0\|\leq\delta\}$ for some $\delta>0$, is $V$ also twice differentiable in $\theta$? If so, can we compute $$ \nabla_{\theta\theta^\top}^2V(\theta) $$ for all $\theta\in\mathrm{Int}B_\delta(\theta_0)$? If the hessian doesn't necessarily exist, what additional conditions should be imposed?

While a general solution to the problem is certainly welcome, for my particular application it is sufficient to handle the case when $c$ is a constant function and both $A(\theta),b(\theta)$ are linear functions of $\theta$.

$\endgroup$

0

You must log in to answer this question.

Start asking to get answers

Find the answer to your question by asking.

Ask question

Explore related questions

See similar questions with these tags.